LSAT and Law School Admissions Forum

Get expert LSAT preparation and law school admissions advice from PowerScore Test Preparation.

User avatar
 Dave Killoran
PowerScore Staff
  • PowerScore Staff
  • Posts: 5862
  • Joined: Mar 25, 2011
|
#42385
Complete Question Explanation
(The complete setup for this game can be found here: lsat/viewtopic.php?t=16021)

The correct answer choice is (B)

If Q is assigned to table 1, P and Z must be assigned to table 3:
J97_Game_#1_#4_diagram 1.png
Since the question stem also states that table 1 has three sponsors, the numerical distribution is now fixed at 3-2-2:
J97_Game_#1_#4_diagram 2.png
Since the four remaining unseated sponsors are K, M, L, and V, two of that group must sit at table 1 and the other two must sit at table 2. Because L and V must sit together as a block, it follows that K and M must also sit together as a block. Thus, the KM block and the LV block cannot sit together:
J97_Game_#1_#4_diagram 3.png
This information is sufficient to prove answer choice (B) correct.
You do not have the required permissions to view the files attached to this post.

Get the most out of your LSAT Prep Plus subscription.

Analyze and track your performance with our Testing and Analytics Package.